Difference between revisions of "2014 AMC 12B Problems/Problem 9"

(Solution)
(Solution)
Line 15: Line 15:
  
 
Note that by the pythagorean theorem, <math>AC=5</math>.  Also note that <math>\angle CAD</math> is a right angle because <math>\triangle CAD</math> is a right triangle.  The area of the quadrilateral is the sum of the areas of <math>\triangle ABC</math> and <math>\triangle CAD</math> which is equal to  
 
Note that by the pythagorean theorem, <math>AC=5</math>.  Also note that <math>\angle CAD</math> is a right angle because <math>\triangle CAD</math> is a right triangle.  The area of the quadrilateral is the sum of the areas of <math>\triangle ABC</math> and <math>\triangle CAD</math> which is equal to  
<cmath>\frac{3*4}{2} + \frac{5*12}{2} = 6 + 30 = \boxed{\textbf{(B)}\ 36}</cmath>
+
<cmath>\frac{3\times4}{2} + \frac{5\times12}{2} = 6 + 30 = \boxed{\textbf{(B)}\ 36}</cmath>

Revision as of 00:11, 21 February 2014

Problem

Convex quadrilateral $ABCD$ has $AB=3$, $BC=4$, $CD=13$, $AD=12$, and $\angle ABC=90^{\circ}$, as shown. What is the area of the quadrilateral?

[asy] pair A=(0,0), B=(-3,0), C=(-3,-4), D=(48/5,-36/5); draw(A--B--C--D--A);  label("$A$",A,N); label("$B$",B,NW); label("$C$",C,SW); label("$D$",D,E); draw(rightanglemark(A,B,C,25)); [/asy]

$\textbf{(A)}\ 30\qquad\textbf{(B)}\ 36\qquad\textbf{(C)}\ 40\qquad\textbf{(D)}\ 48\qquad\textbf{(E)}\ 58.5$

Solution

Note that by the pythagorean theorem, $AC=5$. Also note that $\angle CAD$ is a right angle because $\triangle CAD$ is a right triangle. The area of the quadrilateral is the sum of the areas of $\triangle ABC$ and $\triangle CAD$ which is equal to \[\frac{3\times4}{2} + \frac{5\times12}{2} = 6 + 30 = \boxed{\textbf{(B)}\ 36}\]